Difference between revisions of "2021 JMPSC Sprint Problems/Problem 1"
(One intermediate revision by one other user not shown) | |||
Line 12: | Line 12: | ||
<math>\frac{1}{3}+\frac{1}{6}+\frac{1}{9}=\frac{6}{18}+\frac{3}{18}+\frac{2}{18}=\frac{11}{18}</math> and <math>3+6+9=18</math>, so the answer is <math>\frac{11}{18}\cdot18=\boxed{11}</math>. | <math>\frac{1}{3}+\frac{1}{6}+\frac{1}{9}=\frac{6}{18}+\frac{3}{18}+\frac{2}{18}=\frac{11}{18}</math> and <math>3+6+9=18</math>, so the answer is <math>\frac{11}{18}\cdot18=\boxed{11}</math>. | ||
+ | == Solution 3 == | ||
+ | <cmath>3 \left(\frac{1}{3}\right) + 3 \left(\frac{1}{6}\right) + 3 \left(\frac{1}{9}\right)=1+\frac{1}{2}+\frac{1}{3}=\frac{11}{6}</cmath><cmath>6 \left(\frac{1}{3}\right) + 6 \left(\frac{1}{6}\right) + 6 \left(\frac{1}{9}\right)=2+1+\frac{2}{3}=\frac{22}{6}</cmath><cmath>9 \left(\frac{1}{3}\right) + 9 \left(\frac{1}{6}\right) + 9 \left(\frac{1}{9}\right)=3+\frac{3}{2}+1=\frac{33}{6}</cmath> | ||
+ | Therefore, the answer is <math>\frac{11}{6}+\frac{22}{6}+\frac{33}{6}=11</math> | ||
+ | |||
+ | - kante314 - | ||
+ | |||
+ | == Solution 4 == | ||
+ | |||
+ | ''Solution by pog'' | ||
+ | |||
+ | Recall that, for any nonzero <math>a</math>, <math>b</math>, <math>c</math>, we have that <math>\dfrac1a+\dfrac1b+\dfrac1c = \dfrac{\sum_{\text{sym}}ab}{abc}</math>. By Vieta's, we see that <math>a</math>, <math>b</math>, and <math>c</math> are the roots of the monic cubic polynomial <cmath>Q(x) = x^3 - px^2 + \left(\sum_{\text{sym}}ab\right) x - abc,</cmath> where <math>p</math> is an arbitrary constant equal to <math>a + b + c</math>. | ||
+ | |||
+ | In the given equation, we have that <math>a = 3</math>, <math>b = 6</math>, and <math>c = 9</math>, so <math>p = 3 + 6 + 9 = 18</math> and <math>abc = 3 \cdot 6 \cdot 9 = 162</math>, for <cmath>Q(x) = x^3 - 18x^2 + \left(\sum_{\text{sym}}ab\right) x - 162.</cmath> (Finding the former of the latter two coefficients would lead to bashy casework, which we avoid in this extremely elegant solution.) | ||
+ | |||
+ | Since our cubic has roots <math>3</math>, <math>6</math>, and <math>9</math>, the discriminant of our monic cubic polynomial <math>A^2B^2 + 18ABC - 4B^3 - 4A^3C - 27C^2</math> (where <math>A</math>, <math>B</math>, and <math>C</math> are the latter three coefficients of <math>Q(x)</math>, respectively--do not confuse them for the roots of our polynomial equal to <math>a=3</math>, <math>b=6</math>, and <math>c=9</math>, respectively) must not be equal to <math>0</math>, or our cubic will have a double root, which it clearly does not (proof here is left to the reader). | ||
+ | |||
+ | Thus, substitution of the coefficients into our discriminant gives that <math>324 \cdot \left(\sum_{\text{sym}}ab\right)^2 + 324 \left(\sum_{\text{sym}}ab\right) \cdot 162 - 4\left(\sum_{\text{sym}}ab\right)^3 - 4 \cdot 18^3 \cdot 162 - 27 \cdot 162^2</math> must not be equal to <math>0</math>. Letting <math>\sum_{\text{sym}}ab = n</math>, expanding gives that <math>324n^2 + 52488n - 4n^3 - 3779136 - 708588 = -4n^3 + 324n^2 + 52488n - 4487724</math> is not equal to <math>0</math>. Thus <math>n</math> must not be a root of our cubic, so from here we apply the cubic formula to find that <math>n</math> is not equal to (approximately) <math>-115.853202973303</math>, <math>96.4974047104675</math>, or <math>100.355798262835</math>. | ||
+ | |||
+ | Recall that, from earlier, our answer is equal to <math>\left(\dfrac13+\dfrac16+\dfrac19\right)(3 + 6 + 9) = \left(\dfrac1a+\dfrac1b+\dfrac1c\right)(a+b+c) = \left(\dfrac{\sum_{\text{sym}}ab}{162}\right)(18) = \dfrac{n}{162} \cdot 18 = \dfrac{n}{9}</math>. By bounding, <math>n</math> is between the aforementioned values of <math>96.4974047104675</math> and <math>100.355798262835</math>, so since it must be a multiple of <math>9</math> (all answers in the Junior Mathematicians' Problem Solving Competition are integers), it is equal to <math>99</math> and the requested answer is equal to <math>\dfrac{99}{9} = \boxed{11}</math>. This corroborates with jasperE3's strict bounding. <math>\blacksquare</math> | ||
+ | |||
+ | <div style='text-align: center;'>'''Figure 1:''' Final diagram from AoPS user ''awang11'': | ||
+ | [[File:Screen Shot 2021-07-12 at 10.25.44 AM.png||400px]]</div> | ||
==See also== | ==See also== |
Latest revision as of 09:53, 12 July 2021
Problem
Compute .
Solution
Solving the right side gives . Distributing into the left side gives , so the answer is .
Solution 2
and , so the answer is .
Solution 3
Therefore, the answer is
- kante314 -
Solution 4
Solution by pog
Recall that, for any nonzero , , , we have that . By Vieta's, we see that , , and are the roots of the monic cubic polynomial where is an arbitrary constant equal to .
In the given equation, we have that , , and , so and , for (Finding the former of the latter two coefficients would lead to bashy casework, which we avoid in this extremely elegant solution.)
Since our cubic has roots , , and , the discriminant of our monic cubic polynomial (where , , and are the latter three coefficients of , respectively--do not confuse them for the roots of our polynomial equal to , , and , respectively) must not be equal to , or our cubic will have a double root, which it clearly does not (proof here is left to the reader).
Thus, substitution of the coefficients into our discriminant gives that must not be equal to . Letting , expanding gives that is not equal to . Thus must not be a root of our cubic, so from here we apply the cubic formula to find that is not equal to (approximately) , , or .
Recall that, from earlier, our answer is equal to . By bounding, is between the aforementioned values of and , so since it must be a multiple of (all answers in the Junior Mathematicians' Problem Solving Competition are integers), it is equal to and the requested answer is equal to . This corroborates with jasperE3's strict bounding.
See also
The problems on this page are copyrighted by the Junior Mathematicians' Problem Solving Competition.